B & C
I thought B looked good, then I read C and it matched my anticipation and I felt better about it ...
AndrewArabie on February 23, 2023
  • September 2006 LSAT
  • SEC4
  • Q21
1
Reply
Why A?
The stimulus first discusses replacing saturated fat with unsaturated fat. Then it discusses j...
JackM on January 15, 2023
  • September 2006 LSAT
  • SEC4
  • Q9
4
Replies
Why is then answer D?
Why is D the correct answer and B incorrect? Thanks in advance!
ginakd on June 26, 2022
  • September 2006 LSAT
  • SEC4
  • Q15
1
Reply
answer choice E
E seems somewhat appealing even though the answer is C. Can you provide an example of answer E? W...
nicolebet on January 25, 2022
  • September 2006 LSAT
  • SEC4
  • Q6
1
Reply
B?
Why is B incorrect and why is E correct?
#JW on July 29, 2021
  • September 2006 LSAT
  • SEC4
  • Q5
2
Replies
Help
Could you please go over each answer choices and how we arrive at the correct answer?
yckim2180 on May 20, 2021
  • September 2006 LSAT
  • SEC4
  • Q19
1
Reply
Breakdown
Can I get a breakdown of this question, please? Thank you!
tkj97 on May 28, 2020
  • September 2006 LSAT
  • SEC4
  • Q10
1
Reply
Why is C right?
I still don't get why C is helpful. The argument doesn't say you have to be significantly affec...
Hunter on January 11, 2020
  • September 2006 LSAT
  • SEC4
  • Q23
1
Reply
Difference between B & C
Comparable amount of items are the operative words that broke the tie between B & C. B says that ...
bingolawyer on December 3, 2019
  • September 2006 LSAT
  • SEC4
  • Q16
1
Reply
I got D as the right answer
But I had a hard time eliminating C. Could you explain how to rule out C? Thank you
GLEE on December 20, 2018
  • September 2006 LSAT
  • SEC4
  • Q14
1
Reply
I don't understand why C is the answer
Why is c the answer is it not weakening the argument? also, are the answers for all the prep exam...
maisafaneh on September 29, 2018
  • September 2006 LSAT
  • SEC4
  • Q23
2
Replies
PT 50, S4, Q23
I narrowed it down to C and E and chose E. I read the previous thread on why C is correct. Could ...
smilde11 on August 15, 2018
  • September 2006 LSAT
  • SEC4
  • Q23
1
Reply
Help
Why isn't the answer e?
Claudia on March 13, 2017
  • September 2006 LSAT
  • SEC4
  • Q18
1
Reply